You are on page 1of 38

Data

Sufficiency

1). If p is a positive odd integer, what is the remainder when p is divided by 4 ?

I. When p is divided by 8, the remainder is 5.


II. p is the sum of the squares of two positive integers.

Explanation:

Statement I is sufficient:

When p is divided by 8, the remainder is 5 --> p=8q+5=(8q+4)+1=4(2q+1)+1 --> so the remainder


upon division of p by 4 is 1 (since first term is divisible by 4 and second term yields remainder of 1
upon division by 4).

Sufficient.

Statement II is sufficient:

p is the sum of the squares of two positive integers --> since p is an odd integer then one of the
integers must be even and another odd

p=(2n)^2+(2m+1)^2=4n^2+4m^2+4m+1=4(n^2+m^2+m)+1 --> the same way as above: the


remainder upon division of p by 4 is 1 (since first term is divisible by 4 and second term yields
remainder of 1 upon division by 4).

Sufficient.

So the Answer: D.

2). Is the product of a certain pair of integers even?

I. The sum of the integers is odd.


II. One of the integers is even and the other is odd.

A. Statement I ALONE is sufficient, but statement II alone is not sufficient to answer the question
asked.
B. Statement II ALONE is sufficient, but statement I alone is not sufficient to answer the question
asked.
C. BOTH statements I and II, TOGETHER are sufficient to answer the question asked, but NEITHER
statement ALONE is sufficient to answer the question asked.
D. EACH statement ALONE is sufficient to answer the question asked.
E. Statements I and II TOGETHER are NOT sufficient to answer the question asked, and additional
data specific to the problem are needed.

Explanation:

Statement I is sufficient: The only way the sum of two integers can be odd is if one is even and one is
odd. This would yield an even product. The answer to the prompt question is "yes." This statement,
alone and by itself, is sufficient.

1

Statement II is sufficient: One even, one odd: again, an even product. The answer to the prompt
question is "yes." This statement, alone and by itself, is sufficient.

Each statement is sufficient on its own.

So the Answer is D.

3). What is the hundredths digit of the decimal Z?

I. The tenths digit of 100z is 2


II. The units digit of 1,000z is 2

A. Statement I ALONE is sufficient, but statement II alone is not sufficient to answer the question
asked.
B. Statement II ALONE is sufficient, but statement I alone is not sufficient to answer the question
asked.
C. BOTH statements I and II, TOGETHER are sufficient to answer the question asked, but NEITHER
statement ALONE is sufficient to answer the question asked.
D. EACH statement ALONE is sufficient to answer the question asked.
E. Statements I and II TOGETHER are NOT sufficient to answer the question asked, and additional
data specific to the problem are needed.

Explanation:

Let’s say z=a.bcd. The hundredths digit of z would be the value of “c”.

So the question is c=?



Statement I is insufficient: The tenths digit is 100z is 2 --> 100z=100∗a.bcd=abc.d --> the tenths digit
of 100z is the value of “d”.

So, d=2. Not sufficient to calculate “c”.



Statement II is insufficient: The units digit of 1,000z is 2 --> 1000z=1000∗a.bcd=abcd --> the units
digit of 1000z is the value of “d”. So d=2. Not sufficient to calculate “c”.

Together I and II, No new info, only the value of “d” is known. Not sufficient.

So the Answer is E.

2

4). If x and y are positive integers, is the product xy even?

I. 5x - 4y is even
II. 6x + 7y is even

A. Statement I ALONE is sufficient, but statement II alone is not sufficient to answer the question
asked.
B. Statement II ALONE is sufficient, but statement I alone is not sufficient to answer the question
asked.
C. BOTH statements I and II, TOGETHER are sufficient to answer the question asked, but NEITHER
statement ALONE is sufficient to answer the question asked.
D. EACH statement ALONE is sufficient to answer the question asked.
E. Statements I and II TOGETHER are NOT sufficient to answer the question asked, and additional
data specific to the problem are needed.

Explanation:

xy is even if x is even or y is even or both are even.

Statement I is sufficient: 5x - 4y = even

4y must be even. So 5x must be even. Since 5 is odd, x must be even. Sufficient.

Statement II is sufficient: 6x+7y = even

6x must be even. So 7y must be even, thus y must be even. Sufficient.

So the Answer is D.

5). If n is a three-digit positive integer, what is the sum of the digits of n ?

I. The hundreds digit of n is 3 times the units digit.


II. The hundreds digit of n is 3 more than the tens digit.

A. Statement I ALONE is sufficient, but statement II alone is not sufficient to answer the
question asked.
B. Statement II ALONE is sufficient, but statement I alone is not sufficient to answer the
question asked.
C. BOTH statements I and II, TOGETHER are sufficient to answer the question asked, but
NEITHER statement ALONE is sufficient to answer the question asked.
D. EACH statement ALONE is sufficient to answer the question asked.
E. Statements I and II TOGETHER are NOT sufficient to answer the question asked, and
additional data specific to the problem are needed.

3

Explanation:

Let the three digit integer be abc => 100a+10b+c



Statement I is insufficient: a=3c, no info about b insufficient

Statement II is insufficient:=> a=3+b, no info about c insufficient



Together I and II,

3+b = 3c => Let c = 1, then B = 0, a=3 so the number is 301
Let c = 2, then b = 3, a=6 so the number is 632

Still not sufficient

So the Answer is E.

6). If k is a positive integer, is k the square of an integer?

I. k is divisible by 4.
II. k is divisible by exactly four different prime numbers.

F. Statement I ALONE is sufficient, but statement II alone is not sufficient to answer the
question asked.
G. Statement II ALONE is sufficient, but statement I alone is not sufficient to answer the
question asked.
H. BOTH statements I and II, TOGETHER are sufficient to answer the question asked, but
NEITHER statement ALONE is sufficient to answer the question asked.
I. EACH statement ALONE is sufficient to answer the question asked.
J. Statements I and II TOGETHER are NOT sufficient to answer the question asked, and
additional data specific to the problem are needed.

Explanation:

Statement I is insufficient: k is divisible by 4. If k=4 answer is YES, but if k=8 answers is NO. Not
sufficient.

Statement II is insufficient: k is divisible by exactly four different prime numbers.

We don't know the powers of these primes, so if k=2^2∗3∗5∗7 the answer is NO, but if
k=(2^2∗3∗5∗7)^2 the answer is YES (k equals to square of some integer). Not sufficient.

Together I and II, Again if k=2^2∗3∗5∗7 the answer is NO, but if k=(22∗3∗5∗7)^2 the answer is YES.
Not sufficient.

So the Answer is E.

4

7). What is the value of the positive integer m?

I. When m is divided by 6, the remainder is 3.


II. When 15 is divided by m, the remainder is 6.

A. Statement I ALONE is sufficient, but statement II alone is not sufficient to answer the
question asked.
B. Statement II ALONE is sufficient, but statement I alone is not sufficient to answer the
question asked.
C. BOTH statements I and II, TOGETHER are sufficient to answer the question asked, but
NEITHER statement ALONE is sufficient to answer the question asked.
D. EACH statement ALONE is sufficient to answer the question asked.
E. Statements I and II TOGETHER are NOT sufficient to answer the question asked, and
additional data specific to the problem are needed.

Explanation:

Statement I is insufficient: When m is divided by 6, the remainder is 3.


m = 6a + 3
m can take multiple values.
Clearly insufficient

Statement II is sufficient: When 15 is divided by m, the remainder is 6.

15 = mq + 6 ---> mq = 9

as m is a positive integer

(m,q) = (1,9)
(m,q) = (3,3)
(m,q) = (9,1)

Remainder = 6 (given)

0<= remainder < divisor(m)

Only 9 satisfies.

Clearly B is sufficient.

So the answer is B.

5

8). Is the integer n odd?

I. n is divisible by 3
II. 2n is divisible by twice as many positive integers as n

A. Statement I ALONE is sufficient, but statement II alone is not sufficient to answer the
question asked.
B. Statement II ALONE is sufficient, but statement I alone is not sufficient to answer the
question asked.
C. BOTH statements I and II, TOGETHER are sufficient to answer the question asked, but
NEITHER statement ALONE is sufficient to answer the question asked.
D. EACH statement ALONE is sufficient to answer the question asked.
E. Statements I and II TOGETHER are NOT sufficient to answer the question asked, and
additional data specific to the problem are needed.

Explanation:

Statement I is insufficient:

n is divisible by 3. Clearly not sufficient. Consider n = 3 and n = 6.



Statement II is sufficient: 2n is divisible by twice as many positive integers as n.

Remember:
When odd number n is doubled, 2n has twice as many factors as n.
That's because odd number has only odd factors and when we multiply n by two we remain all these
odd factors as divisors and adding exactly the same number of even divisors, which are odd*2.
Sufficient.

So the Answer is B.

9). If x and y are integers, is the value of x(y + 1) even?

I. x and y are prime numbers.


II. y > 7

A. Statement I ALONE is sufficient, but statement II alone is not sufficient to answer the
question asked.
B. Statement II ALONE is sufficient, but statement I alone is not sufficient to answer the
question asked.
C. BOTH statements I and II, TOGETHER are sufficient to answer the question asked, but
NEITHER statement ALONE is sufficient to answer the question asked.
D. EACH statement ALONE is sufficient to answer the question asked.
E. Statements I and II TOGETHER are NOT sufficient to answer the question asked, and
additional data specific to the problem are needed.

6

Explanation:

x(y + 1) will be even if x is even or/and y is odd.

Statement I is insufficient: x and y are prime numbers. If x is ANY odd prime and y is an even prime,
so 2, then x(y + 1) = odd*(even+1) = odd*odd=odd but in all other cases x(y + 1) = even. Not
sufficient.

Statement II is insufficient: y > 7. Clearly insufficient.

Together I and II, Since y = prime > 7, then y = odd, thus x(y + 1) = x(odd + 1) = x*even = even.
Sufficient.

So the Answer is C.

10). If x is positive, is x > 3?

I. (x - 1)^2 > 4
II. (x - 2)^2 > 9

A. Statement I ALONE is sufficient, but statement II alone is not sufficient to answer the
question asked.
B. Statement II ALONE is sufficient, but statement I alone is not sufficient to answer the
question asked.
C. BOTH statements I and II, TOGETHER are sufficient to answer the question asked, but
NEITHER statement ALONE is sufficient to answer the question asked.
D. EACH statement ALONE is sufficient to answer the question asked.
E. Statements I and II TOGETHER are NOT sufficient to answer the question asked, and
additional data specific to the problem are needed.

Explanation:

Statement I is sufficient: (x - 1)^2 > 4 --> (x+1)(x−3)>0 --> roots are -1 and 3. Now, ">" sign indicates
that the solution lies to the left of a smaller root and to the right of the larger root: x<−1 or x>3.
Since given that x is positive then only one range is valid: x>3. Sufficient.

Statement II is sufficient: (x - 2)^2 > 9 --> (x+1)(x−5)>0 --> roots are -1 and 5. Again, ">" sign indicates
that the solution lies to the left of a smaller root and to the right of the larger root: x<−1 or x>5.
Since given that x is positive then only one range is valid: x>5. Sufficient.

So the Answer is D.

7

11). If ax + b = 0, is x > 0 ?

I. a + b > 0
II. a - b > 0

A. Statement I ALONE is sufficient, but statement II alone is not sufficient to answer the
question asked.
B. Statement II ALONE is sufficient, but statement I alone is not sufficient to answer the
question asked.
C. BOTH statements I and II, TOGETHER are sufficient to answer the question asked, but
NEITHER statement ALONE is sufficient to answer the question asked.
D. EACH statement ALONE is sufficient to answer the question asked.
E. Statements I and II TOGETHER are NOT sufficient to answer the question asked, and
additional data specific to the problem are needed.

Explanation:

Given: b=−ax. Question: is x>0

Statement I is insufficient: a+b>0 --> a−ax>0 --> a(1−x)>0 --> either a>0 and 1−x>0, so x<1 OR a<0 and
1−x<0, so x>1. Not sufficient.

Statement II is insufficient: a−b>0 --> a+ax>0 --> a(1+x)>0 --> either a>0 and 1+x>0, so x>−1 OR a<0
and 1+x<0, so x<−1. Not sufficient.

Together I and II, Sum (1) and (2) (we can do this as the signs of these inequalities are in the same
direction) --> (a+b)+(a−b)>0 --> a>0, so we have first range from (1): x<1 and first case from (2): x>−1
--> −1<x<1, so x may or may not be negative. Not sufficient.

So the Answer is E.

12). Is √(x−3)^2=3−x?

I. x≠3
II. −x|x|>0

A. Statement I ALONE is sufficient, but statement II alone is not sufficient to answer the
question asked.
B. Statement II ALONE is sufficient, but statement I alone is not sufficient to answer the
question asked.
C. BOTH statements I and II, TOGETHER are sufficient to answer the question asked, but
NEITHER statement ALONE is sufficient to answer the question asked.
D. EACH statement ALONE is sufficient to answer the question asked.
E. Statements I and II TOGETHER are NOT sufficient to answer the question asked, and
additional data specific to the problem are needed.

8

Explanation:

Is (x−3)^2=3−x?

Remember: √x^2=|x|. Why?

Couple of things:

The point here is that square root function cannot give negative result: which means that

√(some expression) ≥0.

So √(x^2) ≥0. But what does √x^2 equal to?

Let's consider following examples:

If x=5 --> √x^2=√25=5=x=positive;

If x=−5 --> √x^2=√25=5=−x=positive.

So we got that:

√x^2=x, if x≥0;

√x^2=−x, if x<0.

What function does exactly the same thing? The absolute value function! That is why √x^2=|x|

Back to the original question:

So √ (x−3)^2=|x−3| and the question becomes is: |x−3|=3−x?

When x>3, then RHS (right hand side) is negative, but LHS (absolute value) is never negative, hence
in this case equations doesn't hold true.

When x≤3, then LHS=|x−3|=−x+3=3−x=RHS, hence in this case equation holds true.

Basically question asks is x≤3?

Statement I is insufficient: x≠3. Clearly insufficient.

Statement II is sufficient: −x|x|>0, basically this inequality implies that x<0, hence x<3. Sufficient.

So the answer is B.

9

13). Is 1/(a-b) < b-a ?

I. a < b
II. 1 < |a-b|

A. Statement I ALONE is sufficient, but statement II alone is not sufficient to answer the
question asked.
B. Statement II ALONE is sufficient, but statement I alone is not sufficient to answer the
question asked.
C. BOTH statements I and II, TOGETHER are sufficient to answer the question asked, but
NEITHER statement ALONE is sufficient to answer the question asked.
D. EACH statement ALONE is sufficient to answer the question asked.
E. Statements I and II TOGETHER are NOT sufficient to answer the question asked, and
additional data specific to the problem are needed.

Explanation:

Statement I is sufficient: a < b --> we can rewrite this as: a−b<0 so LHS is negative, also we can
rewrite it as: b−a>0 so RHS is positive --> negative<positive. Sufficient.

Statement II is insufficient: 1 < |a - b| --> if a−b=2 (or which is the same b−a=−2) then LHS>0 and
RHS<0 and in this case the answer will be NO if a−b=−2 (or which is the same b−a=2) then LHS<0 and
RHS>0 and in this case the answer will be YES. Not sufficient.

So the Answer is A.

14). Is |x|>| y|?

I. x2 > y2
II. x > y

F. Statement I ALONE is sufficient, but statement II alone is not sufficient to answer the
question asked.
G. Statement II ALONE is sufficient, but statement I alone is not sufficient to answer the
question asked.
H. BOTH statements I and II, TOGETHER are sufficient to answer the question asked, but
NEITHER statement ALONE is sufficient to answer the question asked.
I. EACH statement ALONE is sufficient to answer the question asked.
J. Statements I and II TOGETHER are NOT sufficient to answer the question asked, and
additional data specific to the problem are needed.

10

Explanation:

Just plug in number to disprove:

Statement I is sufficient:

X = -2 and y = -1 answer is Yes to the question.

X = 2 and y = 1 answer is Yes to the question again.

Since it is modulus always you will have yes to the question. So sufficient.

Statement II is insufficient:

X > y

Not sufficient.

X = -1 and y = -2 answer is No to the question.

X = 2 and y = 1 answer is Yes to the question.

So not sufficient.

So the answer is A.

15). If x is a positive number less than 10, is z greater than the average (arithmetic mean) of x and
10?

I. On the number line, z is closer to 10 than it is to x


II. z = 5x

A. Statement I ALONE is sufficient, but statement II alone is not sufficient to answer the
question asked.
B. Statement II ALONE is sufficient, but statement I alone is not sufficient to answer the
question asked.
C. BOTH statements I and II, TOGETHER are sufficient to answer the question asked, but
NEITHER statement ALONE is sufficient to answer the question asked.
D. EACH statement ALONE is sufficient to answer the question asked.
E. Statements I and II TOGETHER are NOT sufficient to answer the question asked, and
additional data specific to the problem are needed.

11

Explanation:

Given: 0<x<10.

Question: is z greater than the average of x and 10? Or:

Is z>(10+x)/2? --> 2z>10+x?



Statement I is sufficient: On the number line, z is closer to 10 than it is to x --> |10−z|<|z−x| --> as z
is closer to 10 than it is to x, then z>x, so |z−x|=z−x --> two cases for 10-z:

A. z≤10 --> |10−z|=10−z --> |10−z|<|z−x| becomes: 10−z<z−x --> 2z>10+x. Answer to the question
YES.

B. z>10 --> in this case 2z>20 and as x<10, then x+10<20, hence 2z>10+x. Answer to the question
YES.

So sufficient.

Statement II is insufficient: z = 5x --> is 2z>10+x? --> is 10x>10+x? --> is x>10/9. We don't know
that. Not sufficient. (we've gotten that if x>10/9, then the answer to the question is YES, but if
0<x≤10/ 9, then the answer to the question is NO.)

So the Answer is A.

16). If m > 0 and n > 0, is (m+x)/(n+x) > m/n?

I. m < n.
II. x > 0.

A. Statement I ALONE is sufficient, but statement II alone is not sufficient to answer the
question asked.
B. Statement II ALONE is sufficient, but statement I alone is not sufficient to answer the
question asked.
C. BOTH statements I and II, TOGETHER are sufficient to answer the question asked, but
NEITHER statement ALONE is sufficient to answer the question asked.
D. EACH statement ALONE is sufficient to answer the question asked.
E. Statements I and II TOGETHER are NOT sufficient to answer the question asked, and
additional data specific to the problem are needed.

12

Explanation:

Statement I is insufficient: m < n. No info about x. Not sufficient.

Statement II is insufficient: x >0. No info about m and n. Not sufficient.

Together I and II, As from the above two statements nominators and denominator of both fractions
are positive, we can cross multiply --> is (m+x)/(n+x)>m/n --> is (m+x)n>(n+x)m --> is mn+xn>mn+xm
--> is x(n−m)>0 --> as x>0 and n>m, then x(n−m)>0 is true. Sufficient.

So the Answer is C.

17). Is (x + 1)/(x - 3) < 0?

I. -1 < x < 1
II. x2 - 4 < 0

A. Statement I ALONE is sufficient, but statement II alone is not sufficient to answer the
question asked.
B. Statement II ALONE is sufficient, but statement I alone is not sufficient to answer the
question asked.
C. BOTH statements I and II, TOGETHER are sufficient to answer the question asked, but
NEITHER statement ALONE is sufficient to answer the question asked.
D. EACH statement ALONE is sufficient to answer the question asked.
E. Statements I and II TOGETHER are NOT sufficient to answer the question asked, and
additional data specific to the problem are needed.

Explanation:

Is (x+1)/(x−3)<0? --> roots are -1 and 3, so 3 ranges: x<−1, −1<x<3 and x>3 --> check extreme value: if
x some very large number then (x+1)/(x−3)=positive/positive>0 --> in the 3rd range expression is
positive, then in 2nd it'll be negative and in 1st it'll be positive again: + - +. So, the range when the
expression is negative is: −1<x<3.

Thus the question basically becomes: is −1<x<3?

Statement I is sufficient: -1 < x < 1. Sufficient.

Statement II is insufficient: x2-4 < 0 --> −2<x<2. Not sufficient.

So the Answer is A.

13

18). If xyz > 0, is x > 0?

I. xy > 0
II. xz > 0

A. Statement I ALONE is sufficient, but statement II alone is not sufficient to answer the
question asked.
B. Statement II ALONE is sufficient, but statement I alone is not sufficient to answer the
question asked.
C. BOTH statements I and II, TOGETHER are sufficient to answer the question asked, but
NEITHER statement ALONE is sufficient to answer the question asked.
D. EACH statement ALONE is sufficient to answer the question asked.
E. Statements I and II TOGETHER are NOT sufficient to answer the question asked, and
additional data specific to the problem are needed.

Explanation:

XYZ > 0 => X > 0 and YZ > 0 OR x < 0 and YZ < 0

Statement I is insufficient:

XY > 0 means x > 0 and Y > 0 OR X < 0 and Y < 0

So (1) is not sufficient

Statement II is insufficient:

XZ > 0 means X > 0 and Z > 0 OR X < 0 and Z < 0

So (2) is not sufficient.

Together I and II,

So if x < 0 then y < 0 and z < 0

=> xyz < 0 which is contrary to what is given in question

=> x > 0

So the Answer is C.

14

19). On the number line, the distance between x and y is greater than the distance between x and z.
Does z lie between x and y on the number line?

I. xyz<0
II. xy< 0

A. Statement I ALONE is sufficient, but statement II alone is not sufficient to answer the
question asked.
B. Statement II ALONE is sufficient, but statement I alone is not sufficient to answer the
question asked.
C. BOTH statements I and II, TOGETHER are sufficient to answer the question asked, but
NEITHER statement ALONE is sufficient to answer the question asked.
D. EACH statement ALONE is sufficient to answer the question asked.
E. Statements I and II TOGETHER are NOT sufficient to answer the question asked, and
additional data specific to the problem are needed.

Explanation:

The distance between x and y is greater than the distance between x and z, means that we can have
one of the following four scenarios:

A. y--------z--x (YES case)

B. x--z--------y (YES case)

C. y--------x--z (NO case)

D. z--x--------y (NO case)

The question asks whether we have scenarios A or B (z lie between x and y ).

Statement I is insufficient: xyz <0 --> either all three are negative or any two are positive and the
third one is negative. We can place zero between y and z in case A (making y negative and x, z
positive), then the answer would be YES or we can place zero between y and x in case C, then the
answer would be NO. Not sufficient.

Statement II is insufficient: xy<0 --> x and y have opposite signs. The same here: We can place zero
between y and x in case A, then the answer would be YES or we can place zero between y and x in
case C, then the answer would be NO. Not sufficient.

Together I and II, Cases A (answer YES) and case C (answer NO) both work even if we take both
statement together, so insufficient.

A. y----0----z--x (YES case) --> xyz<0 and xy<0;

C. y----0----x--z (NO case) --> xyz<0 and xy<0

So the Answer is E.

15

20). Is m+z > 0 ?

I. m-3z > 0
II. 4z-m > 0

A. Statement I ALONE is sufficient, but statement II alone is not sufficient to answer the
question asked.
B. Statement II ALONE is sufficient, but statement I alone is not sufficient to answer the
question asked.
C. BOTH statements I and II, TOGETHER are sufficient to answer the question asked, but
NEITHER statement ALONE is sufficient to answer the question asked.
D. EACH statement ALONE is sufficient to answer the question asked.
E. Statements I and II TOGETHER are NOT sufficient to answer the question asked, and
additional data specific to the problem are needed.

Explanation:

Statement I is insufficient: m - 3z > 0. Insufficient on its own.

Statement II is insufficient: 4z - m > 0. Insufficient on its own.

Together I and II, Remember we can add inequalities with the sign in the same direction -->
m−3z+4z−m>0 --> z>0, so z is positive. From (1) m>3z=positive, so m is positive too (m is more than
some positive number 3z, so it's positive) --> m+z=positive+positive>0. Sufficient.

So the Answer is C.

21). Is |x-y|>|x|-|y|?

I. y < x
II. xy < 0

A. Statement I ALONE is sufficient, but statement II alone is not sufficient to answer the
question asked.
B. Statement II ALONE is sufficient, but statement I alone is not sufficient to answer the
question asked.
C. BOTH statements I and II, TOGETHER are sufficient to answer the question asked, but
NEITHER statement ALONE is sufficient to answer the question asked.
D. EACH statement ALONE is sufficient to answer the question asked.
E. Statements I and II TOGETHER are NOT sufficient to answer the question asked, and
additional data specific to the problem are needed.

16

Explanation:

Probably the best way to solve this problem is plug-in method. Though there are two properties
worth to remember:

1. Always true: |x+y|≤|x|+|y|, note that "=" sign holds for xy≥0 (or simply when x and y have the
same sign);

2. Always true: |x−y|≥|x|−|y|, note that "=" sign holds for xy>0 (so when x and y have the same
sign) and |x|>|y| (simultaneously). (Our case)

So, the question basically asks whether we can exclude "=" scenario from the second property.

Statement I is insufficient: y < x --> we cannot determine the signs of x and y. Not sufficient.

Statement II is sufficient: xy < 0 --> "=" scenario is excluded from the second property, thus
|x−y|>|x|−|y|. Sufficient.

So the Answer is B.

22).


In triangle ABC above, what is the length of side BC?

I. Line segment AD has length 6


II. x = 36

A. Statement I ALONE is sufficient, but statement II alone is not sufficient to answer the
question asked.
B. Statement II ALONE is sufficient, but statement I alone is not sufficient to answer the
question asked.
C. BOTH statements I and II, TOGETHER are sufficient to answer the question asked, but
NEITHER statement ALONE is sufficient to answer the question asked.
D. EACH statement ALONE is sufficient to answer the question asked.
E. Statements I and II TOGETHER are NOT sufficient to answer the question asked, and
additional data specific to the problem are needed

17

Explanation:

As <BDC=<BCD then the BD=BC. Also as <ADB=180-2x (exterior angle) and the sum of the angles of a
triangle is 180 degrees then in triangle ADB we'll have: x+(180-2x)+<ABD=180 --> <ABD=x. Now, we
have that <ABD=x=<DAB so AD=BD --> AD=BD=BC.

Question: BC=?

Statement I is sufficient: Line segment AD has length 6 --> AD=BD=BC=6. Sufficient.

Statement II is insufficient: x = 36 --> we know only angles which is insufficient to get the length of
any line segment.

So the Answer is A.

23).


In the figure above, if x and y are each less than 90 and PS||QR, is the length of segment PQ less
than the length of segment SR ?

I. x > y
II. x+y > 90
A. Statement I ALONE is sufficient, but statement II alone is not sufficient to answer the
question asked.
B. Statement II ALONE is sufficient, but statement I alone is not sufficient to answer the
question asked.
C. BOTH statements I and II, TOGETHER are sufficient to answer the question asked, but
NEITHER statement ALONE is sufficient to answer the question asked.
D. EACH statement ALONE is sufficient to answer the question asked.
E. Statements I and II TOGETHER are NOT sufficient to answer the question asked, and
additional data specific to the problem are needed

18

Explanation:

Statement I is sufficient:

x+y = 180 degrees.

And x < 90 degrees.

Y = 180-x is greater than 90 degrees.

Suppose if x = 60, y becomes 120(angle SRQ is 60 degree, because y+ angleSRQ = 180 degrees).

Since they are parallel. PQ =SR. So it is sufficient:

Statement II is insufficient:

x<y. Suppose if x = 60, y becomes 120(angle SRQ is 60 degree, because y+ angle SRQ = 180 degrees).

Since they are parallel. PQ =SR.

Suppose if x = 60, and y is equal 70(angle SRQ is 110 degree, because y+ angle SRQ = 180 degrees).

Not PQ is not equal to SR.

So not sufficient.

So the answer is A.

24).

In the figure shown, what is the value of x?

I. The length of line segment QR is equal to the length of line segment RS


II. The length of line segment ST is equal to the length of line segment TU

A. Statement I ALONE is sufficient, but statement II alone is not sufficient to answer the question
asked.
B. Statement II ALONE is sufficient, but statement I alone is not sufficient to answer the question
asked.
C. BOTH statements I and II, TOGETHER are sufficient to answer the question asked, but NEITHER
statement ALONE is sufficient to answer the question asked.
D. EACH statement ALONE is sufficient to answer the question asked.
E. Statements I and II TOGETHER are NOT sufficient to answer the question asked, and additional
data specific to the problem are needed

19

Explanation:

x+∠QSR+∠UST=180 (straight line =180) and ∠R+∠T=90 (as PRT is right angle)

Statement I is insufficient: The length of line segment QR is equal to the length of line segment RS -->
triangle QRS is isosceles --> ∠RQS=∠QSR=(180−∠R)/2 (as ∠RQS+∠QSR+∠R=180 --> 2∗∠QSR+∠R=180
--> ∠QSR=(180−∠R)/2). Not sufficient.

Statement II is insufficient: The length of line segment ST is equal to the length of line segment TU --
> triangle UST is isosceles --> ∠SUT=∠UST=(180−∠T)/2. Not sufficient.

Together I and II, x+∠QSR+∠UST=180 --> x+9180−∠R)/2+(180−∠T)/2=180 -->


(x+360−(∠R+∠T))/2=180 --> since ∠R+∠T=90 --> (x+360−90)/2=180 --> x=45. Sufficient.

So the Answer is C.

25). In the xy plane, point P has coordinates (a,b) and point Q has coordinate(c,d). What is the
distance between point Q and point P?

I. b - d = 4
II. a - c = 3

A. Statement I ALONE is sufficient, but statement II alone is not sufficient to answer the question
asked.
B. Statement II ALONE is sufficient, but statement I alone is not sufficient to answer the question
asked.
C. BOTH statements I and II, TOGETHER are sufficient to answer the question asked, but NEITHER
statement ALONE is sufficient to answer the question asked.
D. EACH statement ALONE is sufficient to answer the question asked.
E. Statements I and II TOGETHER are NOT sufficient to answer the question asked, and additional
data specific to the problem are needed.

Explanation:

Distance between point A(x1,y1) and B(x2,y2) can be found by the formula

D=√(x2−x1)2+(y2−y1)2.

So the distance between point P(a,b) and point Q(c,d) would be D=√ (a−c)2+(b−d)2.

Statement I is insufficient: b-d=4 --> don't know a-c. Not sufficient.

Statement II is insufficient: a-c=3 --> don't know b-d. Not sufficient.

Together I and II, We have all necessary info. Sufficient: D=√ (a−c)2+(b−d)2=√(32+42)=5.

So the Answer is C.

20

26). In the xy-coordinate plane, line l and line k intersect at the point (4,3). Is the product of their
slopes negative?

I. The product of the x-intercepts of lines l and k is positive.


II. The product of the y-intercepts of lines l and k is negative.

A. Statement I ALONE is sufficient, but statement II alone is not sufficient to answer the question
asked.
B. Statement II ALONE is sufficient, but statement I alone is not sufficient to answer the question
asked.
C. BOTH statements I and II, TOGETHER are sufficient to answer the question asked, but NEITHER
statement ALONE is sufficient to answer the question asked.
D. EACH statement ALONE is sufficient to answer the question asked.
E. Statements I and II TOGETHER are NOT sufficient to answer the question asked, and additional
data specific to the problem are needed.

Explanation:

We have two lines: yl=m1x+b1 and yk=m2x+b2. The question: is m1∗m2<0?

Lines intersect at the point (4, 3) --> 3=4m1+b1 and 3=4m2+b2

Statement I is insufficient: The product of the x-intersects of lines L and K is positive. Now, one of the
lines can intersect x-axis at 0<x<4 (positive slope) and another also at 0<x<4 (positive slope), so
product of slopes also will be positive BUT it's also possible one line to intersect x-axis at 0<x<4
(positive slope) and another at x>4 (negative slope) and in this case product of slopes will be
negative. Two different answers, hence not sufficient.

But from this statement we can deduce the following: x-intersect is value of x for y=0 and equals to
x=−b/m --> so (−b1/m1)∗(−b2/m2)>0 --> (b1b2/m1m2)>0.

Statement II is insufficient: The product of the y-intersects of lines L and K is negative. Now, one of
the lines can intersect y-axis at 0<y<3 (positive slope) and another at y<0 (positive slope), so product
of slopes will also be positive BUT it's also possible one line to intersect y-axis at y<0 (positive slope)
and another at y>3 (negative slope) and in this case product of slopes will be negative. Two different
answers, hence not sufficient.

But from this statement we can deduce the following: y-intercept is value of y for x=0 and equals to
y=b --> b1∗b2<0.

Together I and II, (b1b2/m1m2)>0 and b1∗b2<0,

As numerator in (b1b2/m1m2)>0 is negative, then denominator m1m2 must also be negative. So


m1m2<0. Sufficient.

So the Answer is C.

21

27). In the xy-plane, line k passes through the point (1, 1) and line m passes through the point (1, -1).
Are the lines k and m perpendicular to each other?

I. Lines k and m intersect at the point (1, -1)
II. Line k intersects the x-axis at the point (1, 0)


A. Statement I ALONE is sufficient, but statement II alone is not sufficient to answer the question
asked.
B. Statement II ALONE is sufficient, but statement I alone is not sufficient to answer the question
asked.
C. BOTH statements I and II, TOGETHER are sufficient to answer the question asked, but NEITHER
statement ALONE is sufficient to answer the question asked.
D. EACH statement ALONE is sufficient to answer the question asked.
E. Statements I and II TOGETHER are NOT sufficient to answer the question asked, and additional
data specific to the problem are needed.

Explanation:

For one line to be perpendicular to another, their slopes must be negative reciprocals of each other
(if slope of one line is m than the slope of the line perpendicular to this line is −1/m). In other words,
the two lines are perpendicular if and only the product of their slopes is −1.

So basically the question is can we somehow calculate the slopes of these lines.

From stem we have one point for each line.

Statement I is insufficient: gives us the second point of line k, hence we can get the slope of this line,
but we still know only one point of line m. Not sufficient.

Statement II is insufficient: again gives the second point of line k, hence we can get the slope of this
line, but we still know only one point of line m. Not sufficient.

Together I and II, we can derive the slope of line k but for line m we still have only one point, hence
we cannot calculate its slope. Not sufficient.

So the Answer is E.

22

28). Are at least 10 percent of the people in Country X who are 65 years old or older employed?

I. In Country X, 11.3% of the population is 65 years old or older


II. In Country X, of the population 65 years old or older, 20 percent of the men and 10 percent
of the women are employed.

A. Statement I ALONE is sufficient, but statement II alone is not sufficient to answer the
question asked.
B. Statement II ALONE is sufficient, but statement I alone is not sufficient to answer the
question asked.
C. BOTH statements I and II, TOGETHER are sufficient to answer the question asked, but
NEITHER statement ALONE is sufficient to answer the question asked.
D. EACH statement ALONE is sufficient to answer the question asked.
E. Statements I and II TOGETHER are NOT sufficient to answer the question asked, and
additional data specific to the problem are needed

Explanation:

Statement I is insufficient: This particular group composes 11.3% of total population. Clearly
insufficient, as no info about employment rates in this group.

Statement II is sufficient: 10% of men in this group and 20% of women in this group are employed.
No matter how many men and women are in this group, more than 10% will be employed. This is
because the weighted average of 2 individual averages (10% and 20%) must lie between these
individual averages, so percent of employed people in this group is between 10% and 20%.
Sufficient.

So the Answer is B.

29). Henry purchased 3 items during a sale. He received a 20 percent discount off the regular price of
the most expensive item and a 10 percent discount off the regular price of each of the other 2 items.
Was the total amount of the 3 discounts greater than 15 percent of the sum of the regular prices of
the 3 items?

I. The regular price of the most expensive item was $50, and the regular price of the next most
expensive item was $20
II. The regular price of the least expensive item was $15.

A. Statement I ALONE is sufficient, but statement II alone is not sufficient to answer the
question asked.
B. Statement II ALONE is sufficient, but statement I alone is not sufficient to answer the
question asked.
C. BOTH statements I and II, TOGETHER are sufficient to answer the question asked, but
NEITHER statement ALONE is sufficient to answer the question asked.
D. EACH statement ALONE is sufficient to answer the question asked.
E. Statements I and II TOGETHER are NOT sufficient to answer the question asked, and
additional data specific to the problem are needed

23

Explanation:

Let the prices be a, b, and c, so that a > b > c.



Basically the questions: is 0.2a+0.1b+0.1c>0.15(a+b+c)? --> is a>b+c?

Statement I is sufficient: The regular price of the most expensive item was $50 and the regular price
of the next most expensive item was $20 --> a=50, b=20, c≤20 (as the second most expensive item
was $20 then the least expensive item, the third one, must be less than or equal to 20). So the
question becomes: is 50>20+c --> is c<30? As we got that c≤20, hence the above is always true.
Sufficient.

Statement II is insufficient: The regular price of the least expensive item was $15. Clearly
insufficient.

So the Answer is A.

30). If a certain charity collected a total of 360 books, videos, and board games, how many videos did
the charity collect?

I. The number of books that the charity collected was 40 percent of the total number of books,
videos, and board games that the charity collected.
II. The number of books that charity collected was 66(2/3) percent of the total number of
videos and board games that charity collected.

A. Statement I ALONE is sufficient, but statement II alone is not sufficient to answer the
question asked.
B. Statement II ALONE is sufficient, but statement I alone is not sufficient to answer the
question asked.
C. BOTH statements I and II, TOGETHER are sufficient to answer the question asked, but
NEITHER statement ALONE is sufficient to answer the question asked.
D. EACH statement ALONE is sufficient to answer the question asked.
E. Statements I and II TOGETHER are NOT sufficient to answer the question asked, and
additional data specific to the problem are needed








24

Explanation:

Given: B + V + G = 360.

Statement I is insufficient: The number of books that the charity collected was 40 percent of the
total number of books, videos, and board games that the charity collected --> B = 2/5*360 --> B =
144. Not sufficient.

Statement II is insufficient: The number of books that charity collected was 66 2/3 percent of the
total number of videos and board games that charity collected --> B = 2/3*(V + G) --> B = 2/3*(360 -
B) --> B = 144. Not sufficient.

Together I and II, We only know that B = 144 and B + V + G = 360 (V + G = 216) --> we cannot solve
for V. Not sufficient.

So the Answer is E.

31). If $1,000 is deposited in a certain bank account and remains in the account along with any
accumulated interest, the dollar amount of interest, I, earned by the deposit in the first n years is
given by the formula I=1,000((1+r/100)^n-1), where r percent is the annual interest rate paid by the
bank. Is the annual interest rate paid by the bank greater than 8 percent?

I. The deposit earns a total of $210 in interest in the first two years
II. (1 + r/100 )^2 > 1.15
A. Statement I ALONE is sufficient, but statement II alone is not sufficient to answer the
question asked.
B. Statement II ALONE is sufficient, but statement I alone is not sufficient to answer the
question asked.
C. BOTH statements I and II, TOGETHER are sufficient to answer the question asked, but
NEITHER statement ALONE is sufficient to answer the question asked.
D. EACH statement ALONE is sufficient to answer the question asked.
E. Statements I and II TOGETHER are NOT sufficient to answer the question asked, and
additional data specific to the problem are needed

Explanation:

Statement I is sufficient: The deposit earns a total of $210 in interest in the first two years --> I=210
and n=2 --> 210=1,000((1+(r/100)^2−1) --> note that we are left with only one unknown in this
equation, r, and we'll be able to solve for it and say whether it's more than 8, so even without actual
solving we can say that this statement is sufficient.

Statement II is insufficient: (1 + r/100 )^2 > 1.15 --> if r=8 then


(1+(r/100))^2=(1+(8/100))^2=(1.08)^2≈1.16>1.15 so, if r is slightly less than 8 (for example 7.99999),
(1+(r/100))^2 will still be more than 1.15. So, this statement is not sufficient to say whether r>8.

So the Answer is A.

25

32). What fraction of this year's graduating students at a certain college are males?

I. Of this year's graduating students, 33 percent of the males and 20 percent of the females
transferred from another college.
II. Of this year's graduating students, 25 percent transferred from another college.

A. Statement I ALONE is sufficient, but statement II alone is not sufficient to answer the
question asked.
B. Statement II ALONE is sufficient, but statement I alone is not sufficient to answer the
question asked.
C. BOTH statements I and II, TOGETHER are sufficient to answer the question asked, but
NEITHER statement ALONE is sufficient to answer the question asked.
D. EACH statement ALONE is sufficient to answer the question asked.
E. Statements I and II TOGETHER are NOT sufficient to answer the question asked, and
additional data specific to the problem are needed

Explanation:

m/(m+f)=?

Obviously taken separately each statement is not sufficient.

Together I and II, # of transferred students from (1) equals to 033m+0.2f and from (2) it equals to
0.25(m+f) --> 0.33m+0.2f=0.25m+0.25f --> m/f=5/8 --> m/(m+f)=5/13. Sufficient.

So the Answer is C.

33). Jason's salary and Karen's salary were each p percent greater in 1998 than in 1995. What is the
value of p?

I. In 1995 Karen's salary was $2,000 greater than Jason's.


II. In 1998 Karen's salary was $2,440 greater than Jason's.

A. Statement I ALONE is sufficient, but statement II alone is not sufficient to answer the
question asked.
B. Statement II ALONE is sufficient, but statement I alone is not sufficient to answer the
question asked.
C. BOTH statements I and II, TOGETHER are sufficient to answer the question asked, but
NEITHER statement ALONE is sufficient to answer the question asked.
D. EACH statement ALONE is sufficient to answer the question asked.
E. Statements I and II TOGETHER are NOT sufficient to answer the question asked, and
additional data specific to the problem are needed

26

Explanation:

Given: j2=j1(1+p/100) and k2=k1(1+p/100). Question: p=?

Statement I is insufficient: k1−j1=2,000. Not sufficient to calculate p.

Statement II is insufficient: k2−j2=2440. Not sufficient to calculate p.

Together I and II, k2−j2=2440=k1(1+p/100)−j1(1+p/100) -->


2440=k1(1+p/100)−j1(1+p/100)=(1+p/100)(k1−j1)=(1+p/100)2,000 --> 2440=(1+p/100)2,000. Sufficient
to calculate p.

Or another way: difference between their salaries increased by 2440-2000=440, which is


440/2000*100=22%, but difference increases proportionally with the salaries, so increase in salary is
also 22%.

So the Answer is C.

34). Martha bought an armchair and a coffee table at an action and sold both items at her store. Her
gross profit from the purchase and sale of the armchair was what percentage greater than her gross
profit from the purchase and sale of the coffee table?

I. Martha paid 10% more for armchair than for the coffee table.
II. Martha sold the armchair for 20% more than she sold the coffee table.

A. Statement I ALONE is sufficient, but statement II alone is not sufficient to answer the
question asked.
B. Statement II ALONE is sufficient, but statement I alone is not sufficient to answer the
question asked.
C. BOTH statements I and II, TOGETHER are sufficient to answer the question asked, but
NEITHER statement ALONE is sufficient to answer the question asked.
D. EACH statement ALONE is sufficient to answer the question asked.
E. Statements I and II TOGETHER are NOT sufficient to answer the question asked, and
additional data specific to the problem are needed

Explanation:

Let the selling price and cost price of an armchair be Sa and Ca respectively; Let the selling price and
cost price of a coffee table be Sc and Cc respectively.

Basically we need to get: (Sa−Ca)/(Sc−Cc).

Statement I is insufficient: Martha paid 10% more for armchair than for the coffee table -->
Ca=1.1∗Cc. Not sufficient to get the ratio.

Statement II is insufficient: Martha sold the armchair for 20% more than she sold the coffee table -->
Sa=1.2∗Sc. Not sufficient to get the ratio.

Together I and II, (Sa−Ca)/(Sc−Cc)=(1.2∗Sc−1.1∗Cc)/Sc−Cc, still not sufficient to get the ratio.

So the Answer is E.

27

Notice though that if the percents in (1) and (2) were the same then the answer would be C, since
we would be able to factor out the percent and then reduce by Sc−Cc. For example if (1) were
Martha paid 20% more for armchair than for the coffee table, then we would have:
9Sa−Ca)/(Sc−Cc)=(1.2∗Sc−1.2∗Cc)/Sc−Cc=1.2∗(Sc−Cc)/(Sc−Cc)=1.2, which would mean that Martha's
gross profit from the armchair was 20% greater than her gross profit from the coffee table. Or simply
if both the cost price and selling price of the armchair were 20% greater than the cost price and
selling price of the coffee table then the profit would also be 20% greater.

35). This morning, a certain sugar container was full. Since then some of the sugar from this
container was used to make cookies. If no other sugar was removed from or added to the container,
by what percent did the amount of sugar in the container decrease?

I. The amount of sugar in the container after making the cookies would need to be increased
by 30 percent to fill the container
II. Six cups of sugar from the container were used to make the cookies

A. Statement I ALONE is sufficient, but statement II alone is not sufficient to answer the
question asked.
B. Statement II ALONE is sufficient, but statement I alone is not sufficient to answer the
question asked.
C. BOTH statements I and II, TOGETHER are sufficient to answer the question asked, but
NEITHER statement ALONE is sufficient to answer the question asked.
D. EACH statement ALONE is sufficient to answer the question asked.
E. Statements I and II TOGETHER are NOT sufficient to answer the question asked, and
additional data specific to the problem are needed

Explanation:

Let original amount of sugar was N

x unit is taken from the jar.

Statement I is sufficient: (3/10)(N−x)=x

N/x=13/3

Statement II is insufficient.

Statement II is insufficient: it doesn't give us any other information. a cup can be 1gm or 20 gm, or
even 1000gm. since we don't have definite value, It is insufficient.

So the answer is A.

28

36). Of the people who attended a workshop, 60 percent were teachers and some of the teachers
were teachers of language arts. What percent of the people who attended the workshop were
teachers of language arts?

I. 200 people attended the workshop.


II. 72 of the teachers who attended the workshop were not teachers of language arts.

A. Statement I ALONE is sufficient, but statement II alone is not sufficient to answer the
question asked.
B. Statement II ALONE is sufficient, but statement I alone is not sufficient to answer the
question asked.
C. BOTH statements I and II, TOGETHER are sufficient to answer the question asked, but
NEITHER statement ALONE is sufficient to answer the question asked.
D. EACH statement ALONE is sufficient to answer the question asked.
E. Statements I and II TOGETHER are NOT sufficient to answer the question asked, and
additional data specific to the problem are needed

Explanation:

Statement I is insufficient: If 200 people attended the workshop, then 120 were teachers and x of
the 120 teachers were teachers of language arts, where x < 120. The required percent is [x/200] X
100. We don’t know x, hence we cannot answer the required percent. Insufficient

Statement II is insufficient: If y people attended the workshop, then 0.6 y were teachers and 0.6 y -
72 were teachers of language arts. The required percent is [(0.6 y - 72)/y] X 100. We don’t know y,
hence we cannot answer the required percent. Insufficient Taking together I and II, we know y =
200, and hence the required percent is [(0.6 y - 72)/y] X 100 can be answered. Sufficient.

So the answer is C.

37). Whenever martin has a restaurant bill with an amount between $10 and $99, he calculates the
dollar amount of the tip as 2times the tens digit of the amount of his bill. If the amount of the
Martin's most recent restaurant bill was between $10 and $99, was the tip calculated by the martin
on this bill greater than 15 percent of the amount of the bill?

I. The amount of the bill was between $15 and $50


II. The tip calculated by the martin was $8

A. Statement I ALONE is sufficient, but statement II alone is not sufficient to answer the
question asked.
B. Statement II ALONE is sufficient, but statement I alone is not sufficient to answer the
question asked.
C. BOTH statements I and II, TOGETHER are sufficient to answer the question asked, but
NEITHER statement ALONE is sufficient to answer the question asked.
D. EACH statement ALONE is sufficient to answer the question asked.
E. Statements I and II TOGETHER are NOT sufficient to answer the question asked, and
additional data specific to the problem are needed

29

Explanation:

Statement I is insufficient: The amount of the bill was between $15 and $50 --> 15<bill<50. Now if
the bill was 20$ then the tip would be 2*2=$4 (2 times tens digit) so more than 0.15*20=$3 (15% of
the bill) but if the bill was 29$ then the tip would still be 2*2=$4 but in this case less than
0.15*29=~$4.5. Not sufficient.

Statement II is sufficient: The tip calculated by the martin was $8 --> Tip=$8 means that:
40<=bill<50 (so that the tens digit of the bill to be 4). Now, even if the bill was exactly $50 (upper
limit), 15% of it would be 0.15*50=$7.5 and it's still less than $8. So the answer to the question is
YES: the tip (8$) was greater than 15% of the bill. Sufficient.

So the Answer is B.

38). Guests at a recent party ate a total of fifteen hamburgers. Each guest who was neither a student
nor a vegetarian ate exactly one hamburger. No hamburger was eaten by any guest who was a
student, a vegetarian, or both. If half of the guests were vegetarians, how many guests attended the
party?

I. The vegetarians attended the party at a rate of 2 students to every 3 non-students, half the
rate for non-vegetarians.
II. 30% of the guests were vegetarian non-students.

Explanation:

We have two categories of people: vegetarians and students. Like any overlapping sets question,
people can be in these groups or not in these groups, so there aren't actually 4 categories to track.
We know that there are a total of 15 hamburgers; we know that anyone neither a vegetarian nor a
student ate 1 hamburger each; we know that anyone who's a vegetarian, student or both ate no
hamburgers.
Accordingly, we know there must be exactly 15 people (15 burgers, 1 burger per person) who are
neither vegetarians nor students.

We're also told that half of the guests are vegetarians; therefore, half the guests are non-
vegetarians.
Q: how many guests were at the party.

Well, we know that:
Total = veg + non veg
or
Total = 2(veg)
So, if we can figure out the number of vegetarians, we can calculate the total number of guests.

(1) The vegetarians attended the party at a rate of 2 students to every 3 non-students, half the rate
for non-vegetarians.

30

This sounds very complicated. Fortunately, this is data sufficiency, so we just need to understand the
kind of information we have, rather than all the details of the information.
We're given the ratios of student vegetarians to non-student vegetarians and the rates for student
non-vegetarians to non-student non-vegetarians.
Since we know that vegetarians = non-vegetarians, we can now use these ratios to calculate the
ratios of all the different types of guests.
Accordingly, we know what portion of the guests are non-student + non-vegetarian. We also know
there are 15 of these silly people. With a part-to-whole ratio and the number that goes along with
the part, we can calculate the whole: Sufficient.

(2) 30% of the guests were vegetarian non-students.
We know that 50% of the guests are vegetarians, so we now know that 20% of the guests were
vegetarian students.
However, we still don't know what % of the guests are students, so there's no way to figure out how
"15" relates to total guests: Insufficient.
(1) is sufficient, (2) isn't... choose (A).

The answer here is A

39). At least 100 students in a school study Japanese. 4% of students who study French also study
Japanese. Do more students study French than Japanese?

I. 16 students study both French and Japanese


II. 10% of students at school who study Japanese also study French.

Explanation:

Let J be the number of students studying Japanese and F be the number of students studying French.
Given information in the main statement:
J >= 100
0.04F study Japanese and French.
Question: Is F > J?


Statement 1: Students studying J and F is 16. Therefore, 0.04F = 16, or
F = 400. This is insufficient, because J could be 100, in which case the answer to the question in the
main stem is Yes, or J = 500, in which case J>F and the answer to the question is NO. INSUFFICIENT.


Statement 2: The algebraic translation of this statement is: 0.1 J = 0.04 F or F = 2.5 J, therefore F > J.
SUFFICIENT.

The answer here is D.

31

40). Does the integer K have a factor p such that 1 < p < K?

I. K > 4!
II. 13! + 2 < k < 13! + 13

Explanation:

A prime number has exactly two factors: itself and 1. By definition, it cannot have a factor p that is
between itself and 1. A non-prime number, on the other hand, that is greater than 1 will have a
factor between itself and 1 (again, by definition).
So I can answer this question if I know whether k is prime.
(1) Just tells me that k>24. There are prime numbers and non-prime numbers greater than 24, so
that's not useful.
(2) For any sum, if the two numbers in that sum have a common factor, that factor will also be a
factor of the sum. E.g., 2 + 4 = 6. 2 is a factor of 2 and 2 is a factor of 4. Therefore, 2 will also be a
factor of 6.

Answer is B.

41). In the xy-plane, at what two points does the graph of y = (x+a)(x+b) intersect the x-axis?

I. a+b = -1
II. The graph intersects the y-axis at (0,-6)

Explanation:

1. Insufficient. All we have it x^2 -x + ab = 0. Cannot solve for x


2. insufficient. y = x^2 + (a+b)x + ab implies ab = -6. Cannot solve for x
Using 1 and 2,
x^2 - x - 6 = 0
x = 3 & -2


So, C is the correct answer

42). A combined of 55 light bulbs are stored in two boxes; of these, a total of 7 are broken. If there
are exactly two broken bulbs in the first box, what is the number of bulbs in the second box that are
not broken?

I. In the first box, the number of bulbs that are not broken is 15 times the number of the
broken bulbs.
II. The total number of bulbs in the first box is 9 more than the total number of bulbs in the
second box.

32

Explanation:

We are asked for the number not broken in the second box. We know there are 55 bulbs in total and
that 7 of them are broken. We know there are 2 broken in the first box, and 5 broken in the second
box, (and that the other 48 remaining bulbs are not broken.)


Statement 1: We will be able to compute the number of not broken in the second box if we know
the total number in the second box...or if we know the total number in the first box...and, finally,
because we know that the total is 55, we will also have sufficiency if we get a special equation
relating the total number in the first box to the total number in the second box.


Statement 2: this is a special equation that will allow us to compute the total number in the second
box. Once we have that, we would simply subtract the 5 broken, and we'd have the number not
broken in the second box. But because this is data sufficiency, instead of actually doing that math,
we would just realize (after/during our analysis) that we COULD do it.
So here's the work we wouldn't actually do:
# in 1st box + # in 2nd box =55 (from question)
# in 1st box = # in 2nd box + 9 (statement 2)
subbing in:
(# in 2nd box + 9) + # in 2nd box =55
solving:
# in 2nd box = 23
Therefore, number of not broken in 2nd box is 23-5= 18

Answer is D.

43). The number N is 4,5H2, the ten's digit being represented by H. What is the value of H?

I. N is divisible by 3.
II. N is divisible by 9.

Explanation:

Statement (1) is insufficient. If the number is divisible by 3, the digits must add up to a multiple of
three. Therefore, 4+5+H+2 must sum to a multiple of 3. That would be true if H=1, H=4, or H=7.

Statement (2) is sufficient: if the number is divisible by 9, the digits must add up to a multiple of 9.
The only multiple of 9 that 4+5+H+2 could sum to is 18, which means that H=7.

The correct choice is (B).

33

44). Working independently at their respective constant rates, machines X and Y took 15 minutes to
fill an order. What fraction of the order was filled by machine X?

I. Working alone at its constant rate, machine X would have taken 60 minutes to fill the order.
II. Working alone at its constant rate, machine Y would have taken 20 minutes to fill the order.

Explanation:

Combined, the machines fill one order in 15 minutes. In those 15 minutes, each of the machines fills
some fraction of the order. Since we know the time (15 minutes), we need to know the rate of
machine X to find the fraction of the order--how much of the job--machine X would fill.

Statement (1) is sufficient: if machine X fills one order in 60 minutes, we can find out how many
orders it could fill in 15 minutes--one-fourth as many, or (1/4) of an order.

Statement (2) is also sufficient. Using the same technique, we can determine that Y would fill (3/4) of
an order in 15 minutes, which leaves (1/4) of the order for machine X.

Choice (D) is correct.

45). The average (arithmetic mean) monthly balance in Company X's petty cash account on any given
date is the average of the losing balances posted on the last business day of each of the past 12
months. On March 6, 1990, the average monthly balance was 692.02. What was the average
monthly balance as of June 23rd, 1990?

I. As of June 23,1990, the total of all closing balances posted on the last business day of each
of the last 12 months was $45.64 less than it had been on March 6, 1990
II. The closing balances posted on the last business days of March, April, and May 1990 were
$145.90, $3000.00 and $725.25 respectively

Explanation:

Let's start by deconstructing the question stem; a good general rule for DS is that the longer the
stem, the more time you should spend thinking about it.
The average monthly balance on March 6, 1990, is the average of the closing balances for March
1989-February 1990.
The average monthly balance on June 23, 1990, is the average of the closing balances for June 1989-
May 1990.
What's the difference between the two? The first includes March, April and May 1989; the second
includes March, April and May 1990 instead.

(1) Gives us the difference between the second year and the first, allowing us to calculate the June
23, 1990, average monthly balance - sufficient.
(2) Gives us March, April and May 1990, which is a good start, but not enough.
If we had the difference between March, April and May 1990 and those same months in 1989, we
could answer the question. However, without info about those months in 1989, we have no idea

34

what the year-over-year change is.

For example, if those 3 months in 1989 had the exact same balances as in 1990, then the answer
would be $692.02.
If those 3 months in 1989 had lower balances, then the answer would be more than $692.02.
If those 3 months in 1989 had higher balances, then the answer would be less than $692.02.
(1) is sufficient, (2) isn't

The answer here is A.

46). How many odd integers are greater than integer X and less than the integer y?

I. There are 12 even integers greater than x and less than y


II. There are 24 integers greater than X and less than Y

Explanation:

Guess this can be better explained using an example. Take 2 series


X=1, Y=26, the series is 1 2 ... 25 26
X=2, Y=27, the series is 2 3 ... 26 27

In each case, the total number of odd integers is the same.
X=1, Y=26, series : 1 2 ... 25 26
Odd integers are 3, 5, 7, 9, 11, 13, 15, 17, 19, 21, 23, 25
X=2, Y=27, series : 2 3 ... 26 27
Odd integers are 3, 5, 7, 9, 11, 13, 15, 17, 19, 21, 23, 25


The answer here is B.

47). The sequence s1, s2, s3,.....sn,...is such that Sn= (1/n) - (1/(n+1)) for all integers n>=1. If k is a
positive integer, is the sum of the first k terms of the sequence greater than 9/10?

I. k > 10
II. k < 19

Explanation:

S1 = 1 - (1/2)
S2 = (1/2) - (1/3)
...
Sn = 1/n - (1/(n+1)
Sum it up.
you are left with 1 - (1/n+1)

35



Is 1- (1/n+1) > 9/10 ?
or 1/10 > 1/(n+1)
or n +1 >10
or n > 9
In other words, is #terms > 9
(1) says #terms > 10, sufficient
(2) says #terms < 19. Here #terms can be 7, or 17. Insufficient.

48). What is the value of t^3 - m^3?

I. t^2 - m^2 = 18
II. t - m = 2

Explanation:

Within the scope of GMAT math, there is no way to simplify t^3 - m^3, so in order to answer the
question, we'll need the values of both t and m.

Statement (1) is insufficient. We can factor and find that (t + m)(t - m) = 18, but that doesn't give us
what we need.

Statement (2) is also insufficient. Two variables and one equation isn't enough to solve for the
variables. Taken together, the statements are sufficient. If t - m = 2, we can substitute that into the
factored version of (1):
(t + m)(2) = 18
t + m = 9
Now we have two equations and two variables:
t + m = 9
t - m = 2
Add the equations:
2t = 11
t = 5.5
From there, we can find m and answer the question. No need to do the rest of the math.

The answer is (C).

36

49). If production on line A increased 5% from 2006 to 2007, and if production on line B increased
10% in the same period, how many units did line A produce in 2006?

I. The two lines combined produced 100,000 units total in 2006.


II. The two lines combined produced 107,500 units total in 2007.

Explanation:

The question is asking for how many units line A produced in 2006
from stmt1:
A + B = 100,000
Since there is no relation between the variables A and B, we cannot solve for A, insufficient
from stmt2:
1.05A + 1.1B = 107,500
Again, no relation between variables A and B, so we cannot solve for A, insufficient
Looking at both statements together:
We have two equations relating A and B. Looking at both equations you can already see that the two
equations are different and can be used to solve for variable A. Both statement together are
sufficient to solve the problem, answer is C.
If you want to find the actual numerical answer (even though unnecessary)
I would multiply stmt 1 by 1.1, so:
1.1A + 1.1B = 110,000
Take this statement and subtract stmt 2:
1.1A + 1.1B = 110,000
-1.05A - 1.1B = -107,500
and you get 0.05A = 2,500
Solve for variable A, => A = 50,000

The answer here is C.

37

50). After the first term, each term in a sequence is five times greater than half the preceding term.
If x is the first term of the sequence, and x does not equal zero, what is the value of the fourth term
minus the second term an integer?

I. x is a multiple of 12.
II. x is a multiple of 56.

Explanation:

The statement definitely means that the next term is 2.5 times the previous term. But looking at it
that way makes it more complicated. I think the answer should be "b" because:
The difference between the 4th term and the 2nd term will be an integer if both the 4th and 2nd
term are integers. Likewise, the next term in the sequence is only an integer if 1/2 of it is in integer.
Thus if x is a multiple of 12, the second term is a multiple of 6, the third term is a multiple of 3, and
the fourth terms is a multiple of 1/2. If x has another factor of 2 in it (let's say its 24), the fourth term
would be an integer. But if x does not have another factor of 2, (let's say it's 36), the fourth term is
not an integer and the second one is. Therefore, we don't have enough info.
But with statement 2, 1st term is multiple of 56, 2nd term is multiple of 28, 3d term is multiple of 14
and 4th term is multiple of 7. Since every multiple of 7 is an integer, we must have an integer minus
an integer, so b alone is sufficient.

Answer is B.

38

You might also like